initial seminar rewrite
authorhackbard <hackbard>
Fri, 23 May 2003 12:35:24 +0000 (12:35 +0000)
committerhackbard <hackbard>
Fri, 23 May 2003 12:35:24 +0000 (12:35 +0000)
ising/ising_slides.tex

index 3e3b247..68f712f 100644 (file)
@@ -1,4 +1,4 @@
-\documentclass[a4,12pt]{seminar}
+\documentclass{seminar}
 
 \usepackage{verbatim}
 \usepackage[german]{babel}
 
 \usepackage{verbatim}
 \usepackage[german]{babel}
@@ -19,7 +19,6 @@
 \graphicspath{{./img/}}
 
 \usepackage{semcolor}           % Seminar colored slides
 \graphicspath{{./img/}}
 
 \usepackage{semcolor}           % Seminar colored slides
-% \usepackage{semhelv}            % Seminar helvetica fonts
 \usepackage{semlayer}           % Seminar overlays
 \usepackage{slidesec}           % Seminar sections and list of slides
 
 \usepackage{semlayer}           % Seminar overlays
 \usepackage{slidesec}           % Seminar sections and list of slides
 
 \tableofcontents
 \end{slide}
 
 \tableofcontents
 \end{slide}
 
+\begin{slide}
 \section{Einf"uhrung}
 \section{Einf"uhrung}
+\end{slide}
 
 
-\subsection{Zustandssumme und ben"otigte Gr"o"sen}
-Unter der kanonischen Zustandssumme versteht man die Summation ueber alle m"oglichen Mikrozust"ande.
+\begin{slide}
+\slideheading{Zustandssumme und ben"otigte Gr"o"sen}
+Zustandssumme: Summation "uber alle m"oglichen Mikrozust"ande
 \[
  Z = \sum_{i=1}^{N}e^{\frac{-E_i}{k_BT}} = \textrm{Sp} \,  e^{-\beta H} \qquad \beta = \frac{1}{k_BT}
 \]
 \[
  Z = \sum_{i=1}^{N}e^{\frac{-E_i}{k_BT}} = \textrm{Sp} \,  e^{-\beta H} \qquad \beta = \frac{1}{k_BT}
 \]
-Sie ist eine fundamentale Gr"o"se in der statistischen Physik. Von ihr k"onnen viele wichtige Gr"o"sen abgeleitet werden.
+Ableitung wichtiger Gr"o"sen:
 \begin{itemize}
 \begin{itemize}
-\item Wahrscheinlichkeit fuer Zustand $i$: $P_i = \frac{1}{Z} e^{-E_i \beta}$
+\item Wahrscheinlichkeit f"ur Zustand $i$: $P_i = \frac{1}{Z} e^{-E_i \beta}$
 \item freie Energie: $F = -k_BT \, \textrm{ln} \, Z$ 
 \item Magnetisierung: $M = - \frac{\partial F}{\partial B}$
 \item magnetische Suszeptibilit"at: $\chi = - \frac{\partial M}{\partial H}$
 \item spezifische W"armekapazit"at: $c = \frac{\partial E}{\partial T}$
 \end{itemize}
 \item freie Energie: $F = -k_BT \, \textrm{ln} \, Z$ 
 \item Magnetisierung: $M = - \frac{\partial F}{\partial B}$
 \item magnetische Suszeptibilit"at: $\chi = - \frac{\partial M}{\partial H}$
 \item spezifische W"armekapazit"at: $c = \frac{\partial E}{\partial T}$
 \end{itemize}
+\end{slide}
 
 
-\subsection{Phasen"uberg"ange}
-Die Phase ist eine m"ogliche Zustandsform eines makroskopischen Systems im thermischen Gleichgewicht. Unterschiedliche Phasen "aussern sich in unterschiedlichen Werten makroskopischer Observablen. Beispiele:
+\begin{slide}
+\slideheading{Phasen"uberg"ange}
+Die Phase ist eine m"ogliche Zustandsform eines makroskopischen Systems im thermischen Gleichgewicht. Unterschiedliche Phasen "au"sern sich in unterschiedlichen Werten makroskopischer Observablen.\\
+Beispiele:
 \begin{itemize}
 \begin{itemize}
-\item Dichte
-\item Magnetisierung
-\item elektrische Leitf"ahigkeit
+\item Dichte ($H_2O$)
+\item Magnetisierung (Nickel)
+\item elektrische Leitf"ahigkeit ($YBa_2Cu_2O_7$)
 \end{itemize}
 \end{itemize}
-Mit einem Phasen"ubergang verbunden ist ein kitischer Bereich einer Variablen, in dem sich die Phase "andert. Man unterscheidet "Ubrg"ange erster Ordnung (diskontinuierlich) und "Ubergaenge zweiter Ordnung (kontinuierlich).
+\end{slide}
+
+\begin{slide}
+Phasen"ubergang verbunden mit kritischen Bereich einer Variablen
 \begin{itemize}
 \item diskontinuierlich: Unstetigkeit in erster Ableitung eines thermodynamischen Potentials
 \item kontinuierlich: Stetigkeit der ersten Ableitung, Unstetigkeit der zweiten Ableitung (Bsp: Magnetisierung - Suszeptibilit"at)
 \begin{itemize}
 \item diskontinuierlich: Unstetigkeit in erster Ableitung eines thermodynamischen Potentials
 \item kontinuierlich: Stetigkeit der ersten Ableitung, Unstetigkeit der zweiten Ableitung (Bsp: Magnetisierung - Suszeptibilit"at)
+% \\
+% \includegraphics[width=10cm,clip,draft=no]{phasen_ue}
 \end{itemize}
 \end{itemize}
+\end{slide}
 
 
-\subsection{Kritische Exponenten}
-In der N"ahe eines Phasen"ubergangs beobachtet man das gewisse physikalische Gr"oe"sen Potenzgesetzen gehorchen. Diese Exponenten beschreiben wie die physikalischen Gr"o"sen nahe $T_C$ divergieren.
+\begin{slide}
+\slideheading{Kritische Exponenten}
+Physikalische Gr"oe"sen gehorchen Potenzgesetzen nahe des kritischen Bereichs
 \begin{itemize}
 \item Magnetisierung $M \sim |T-T_C|^\beta$
 \item spezifische Wa"rmekapazit"at $c \sim |T-T_C|^\alpha$
 \begin{itemize}
 \item Magnetisierung $M \sim |T-T_C|^\beta$
 \item spezifische Wa"rmekapazit"at $c \sim |T-T_C|^\alpha$
-\item Suszeptibilit"at $\chi \sim |T-T_C|^\gamma$
+\item Suszeptibilit"at $\chi \sim |T-T_C|^{-\gamma}$
 \end{itemize}
 Anmerkung:\\
 \end{itemize}
 Anmerkung:\\
-Kritische Exponenten sind zu einem hohen Grad universell, d.h. sie h"angen nur von fundamentalen Parametern wie Dimension, Reichweite der Teilchen-Wechselwirkung und Spindimensionalit"at ab, und nicht vom Modell selbst. Damit lassen sich Universalit"atsklassen definieren.
+Kritische Exponenten universell $\rightarrow$ Abhaengig von Dimension, Reichweite und Struktur der Wechselwirkung $\rightarrow$ Universalit"atsklassen
+\end{slide}
 
 
-\subsection{Idee des Ising Modells}
+\begin{slide}
+\slideheading{Idee des Ising Modells}
+Modell f"ur magnetische Phasen"uberg"ange.\\
+\\
 Modellannahmen: 
 \begin{itemize}
 \item $d$-dimensionales periodisches Gitter, $d=1,2,3$
 Modellannahmen: 
 \begin{itemize}
 \item $d$-dimensionales periodisches Gitter, $d=1,2,3$
@@ -88,31 +104,45 @@ Modellannahmen:
 \]
 \item lokalisierte Momente wechselwirken miteinander, Kopplungskonstante sei $\frac{J_{ij}}{\mu^2}$
 \end{itemize}
 \]
 \item lokalisierte Momente wechselwirken miteinander, Kopplungskonstante sei $\frac{J_{ij}}{\mu^2}$
 \end{itemize}
-Dann lautet die Hamilton-Funktion:
+Hamilton-Funktion:
 \[
  H = - \sum_{(i,j)} J_{ij} S_i S_j - \mu B_0 \sum_i S_i \, \textrm{, mit}
 \]
 \[
 \[
  H = - \sum_{(i,j)} J_{ij} S_i S_j - \mu B_0 \sum_i S_i \, \textrm{, mit}
 \]
 \[
-(i,j) = \textrm{naechste Nachbarn im Gitter,} \qquad \vec{B} = (0,0,B_0)
+(i,j) = \textrm{n"achste Nachbarn im Gitter,} \qquad \vec{B} = (0,0,B_0)
 \]
 \]
+\end{slide}
+
+\begin{slide}
 Man erahnt: ($J > 0$, \, Ferromagnet)
 \begin{itemize}
 Man erahnt: ($J > 0$, \, Ferromagnet)
 \begin{itemize}
-\item $T \to 0$: \\ $\longrightarrow \textrm{Zustand niedrieger Energie} \longrightarrow \textrm{Spins gleich ausgerichtet}$ \\ $\longrightarrow \textrm{hohe Magnetisierung}$
-\item $T \to \infty$: \\ $\longrightarrow \textrm{Zustand hoher Energie} \longrightarrow \textrm{Spins zufaellig ausgerichtet}$ \\ $\longrightarrow \textrm{keine Magnetisierung}$
+\item $T \to 0$: \\ $\longrightarrow \textrm{Zustand niedriger Energie} \longrightarrow \textrm{Spins gleich ausgerichtet}$ \\ $\longrightarrow \textrm{hohe Magnetisierung}$
+\item $T \to \infty$: \\ $\longrightarrow \textrm{Zustand hoher Energie} \longrightarrow \textrm{Spins zuf"allig ausgerichtet}$ \\ $\longrightarrow \textrm{keine Magnetisierung}$
 \end{itemize}
 \end{itemize}
-Unter einer bestimmten Temperatur stellt sich auch ohne Aenderung eines aeusseren Magnetfeldes eine spontane Magnetisierung ein, dies laesst auf einen Phasenuebergang zweiter Ordnung schliessen (Divergenz von $\chi$).
-\\
-\\
+Ergebnis: spontane Magnetisierung unterhalb kritischer Temperatur ohne externes $B$-Feld
+\end{slide}
+
+\begin{slide}
 Molekularfeldn"aherung:\\
 Molekularfeldn"aherung:\\
-Approximation des Ising Modells durch Vernachl"assigung der Spinfluktuationen $S_i-<S_i>)$. Damit kann man den Spin-Wechselwirkungs-Term umschreiben:
+Approximation: Vernachl"assigung der Spinfluktuationen $S_i-<S_i>$\\
+Spin-Wechselwirkungs-Term:
 \[
  S_iS_j = (S_i-m+m)(S_j-m+m)=m^2+m(S_i-m)+m(S_j-m)+(S_i-m)(S_j-m)
 \]
 \[
  S_iS_j = (S_i-m+m)(S_j-m+m)=m^2+m(S_i-m)+m(S_j-m)+(S_i-m)(S_j-m)
 \]
-wobei $m=\frac{1}{N}(sum_i^N S_i)$ die mittlere Magnetisierung pro Spin ist und der letzte Term damit von der Gestalt $(S_i-<S>)(S_j-<S>)$ ist und in der MFN vernachl"assigt wird. Mit der Definition $\sum_j J_{ij} \equiv J^{'} \cdot z \equiv J$, wobei $z$ die Anzahl der n"achsten Nachbarn ist, erhalten wir folgenden Hamiltonian,
+wobei:
+\begin{itemize}
+\item $m=\frac{1}{N}(\sum_i^N S_i)$, mittlere Magnetisierung pro Spin 
+\item $(S_i-<S>)(S_j-<S>)$ in MFN vernachl"assigt
+\item Definition: $\sum_j J_{ij} \equiv J^{'} \cdot z \equiv J$, mit $z$  Anzahl der n"achsten Nachbarn
+\end{itemize}
+\end{slide}
+
+\begin{slide}
+Hamiltonian:
 \[
  H_{MFN} = \frac{1}{2} NJm^2 - (Jm + \mu B_0) \sum_i S_i
 \]
 \[
  H_{MFN} = \frac{1}{2} NJm^2 - (Jm + \mu B_0) \sum_i S_i
 \]
-und Zustandssumme:
+Zustandssumme:
 \[
 \begin{array}{ll}
  Z  & = e^{- \beta \frac{NJm^2}{2}} \sum_{S_1} \ldots \sum_{S_N} \, e^{\beta (Jm + \mu B_0) \sum_i S_i} \\
 \[
 \begin{array}{ll}
  Z  & = e^{- \beta \frac{NJm^2}{2}} \sum_{S_1} \ldots \sum_{S_N} \, e^{\beta (Jm + \mu B_0) \sum_i S_i} \\
@@ -120,38 +150,58 @@ und Zustandssumme:
     & = e^{- \beta \frac{NJm^2}{2}} \Big( 2 \cosh (\beta (Jm + \mu B_0)) \Big)^N
 \end{array}
 \]
     & = e^{- \beta \frac{NJm^2}{2}} \Big( 2 \cosh (\beta (Jm + \mu B_0)) \Big)^N
 \end{array}
 \]
-Damit erhalten wir f"ur die freie Energie und Magnetisierung pro Spin folgendes:
+\end{slide}
+
+\begin{slide}
+freie Energie und Magnetisierung pro Spin:
 \[
 \begin{array}{l}
  g = - \frac{1}{N \beta} \textrm{ln} \, Z = - \frac{1}{2} Jm^2 - \frac{1}{\beta} \textrm{ln} \, \Big(2 \cosh (\beta (Jm + \mu B_0)) \Big) \\
  m = - \Big( \frac{\partial g}{\partial B_0} \Big) = \tanh (\beta (Jm + \mu B_0))
 \end{array}
 \]
 \[
 \begin{array}{l}
  g = - \frac{1}{N \beta} \textrm{ln} \, Z = - \frac{1}{2} Jm^2 - \frac{1}{\beta} \textrm{ln} \, \Big(2 \cosh (\beta (Jm + \mu B_0)) \Big) \\
  m = - \Big( \frac{\partial g}{\partial B_0} \Big) = \tanh (\beta (Jm + \mu B_0))
 \end{array}
 \]
-Legt man nun kein magnetisches Feld $B_0$ an, so hat man eine implizite Bestimmungsgleichung f"ur die Magnetsisierung
+implizite Bestimmungsgleichung f"ur die Magnetsisierung mit $B_0=0$:
 \[
  \tanh (\beta Jm) = m
 \]
 \[
  \tanh (\beta Jm) = m
 \]
-die grafisch diskutiert werden kann. Man findet L"osungen $m \neq 0$ wenn die Anfangssteigung der linken Seite der Gleichung gr"o"ser $1$ ist. F"ur die kritische Temperatur gilt somit $\frac{\partial (\tanh (\beta Jm))}{\partial m} = 1$.
-\\
-\setlength{\unitlength}{2cm}
-\begin{picture}(6,4)(-3,-2)
- \put(0,0){\line(1,1){1}}
- \put(0,0){\line(-1,-1){1}}
- \put(-2.5,0){\vector(1,0){5}}
- \put(2.7,-0.1){$m$}
- \put(0,-1.5){\vector(0,1){3}}
- \multiput(-2.5,1)(0.4,0){13}{\line(1,0){0.2}}
- \multiput(-2.5,-1)(0.4,0){13}{\line(1,0){0.2}}
- \put(0.2,1.4){$f(m)$}
- \qbezier(0,0)(0.6,0.9)(2,0.9640)
- \qbezier(0,0)(-0.6,-0.9)(-2,-0.9640)
-\end{picture}
-\\
-Man findet also einen Phasen"ubergang unabh"angig von der Gitterdimension. Die folgende exakte L"osung des eindimensionalen Isingmodells widerspricht dem, ist jedoch typisch f"ur alle klassischen Theorien (Bsp: Landau-Theorie).
+\end{slide}
+
+\begin{slide}
+\begin{itemize}
+\item L"osung: $m \neq 0 \longleftrightarrow \frac{\partial (\tanh (\beta Jm))}{\partial m} > 1$ bei $m=0$
+\item kritische Temperatur: $\frac{\partial (\tanh (\beta Jm))}{\partial m} = 1$ bei $m=0$
+\end{itemize}
 
 
+% \setlength{\unitlength}{2cm}
+% \begin{picture}(6,4)(-3,-2)
+%  \put(0,0){\line(1,1){1}}
+%  \put(0,0){\line(-1,-1){1}}
+%  \put(-2.5,0){\vector(1,0){5}}
+%  \put(2.7,-0.1){$m$}
+%  \put(0,-1.5){\vector(0,1){3}}
+%  \multiput(-2.5,1)(0.4,0){13}{\line(1,0){0.2}}
+%  \multiput(-2.5,-1)(0.4,0){13}{\line(1,0){0.2}}
+%  \put(0.2,1.4){$f(m)$}
+%  \qbezier(0,0)(0.6,0.9)(2,0.9640)
+%  \qbezier(0,0)(-0.6,-0.9)(-2,-0.9640)
+% \end{picture}
+% \\
+
+\includegraphics[width=08cm,clip,draft=no]{meanfield_mag.eps}
+
+\begin{itemize}
+\item Phasen"ubergang unabh"angig von Gitterdimension
+\item Widerspruch zu exakter $d=1$ L"osung
+\item Typisch f"ur alle klassischen Theorien (Bsp: Landau-Theorie)
+\end{itemize}
+\end{slide}
+
+\begin{slide}
 \section{Loesungen des Ising Modells}
 \section{Loesungen des Ising Modells}
+\end{slide}
 
 
-\subsection{1-dimensionale L"osung}
+\begin{slide}
+\slideheading{L"osung f"ur $d=1$}
 \setlength{\unitlength}{1.5cm}
 \begin{picture}(10,1)
  \thicklines
 \setlength{\unitlength}{1.5cm}
 \begin{picture}(10,1)
  \thicklines
@@ -168,26 +218,35 @@ Man findet also einen Phasen"ubergang unabh"angig von der Gitterdimension. Die f
  \put(6,0){$N$}
 \end{picture} \\
 \\
  \put(6,0){$N$}
 \end{picture} \\
 \\
-Die Hamilton-Funktion in einer Dimension lautet:
+Hamilton-Funktion:
 \[
  H = - \sum_{i=1}^{N} J_{i,i+1} S_i S_{i+1} - \mu B_0 \sum_{i=1}^{N} S_i
 \]
 \[
  H = - \sum_{i=1}^{N} J_{i,i+1} S_i S_{i+1} - \mu B_0 \sum_{i=1}^{N} S_i
 \]
-Annahme: periodische Randbedingungen, $S_{N+1} \equiv S_1$ \, (Translationsinvarianz), $J_{i,i+1} \equiv J$ \\
-Abkuerzung: $K = \beta J$, $h = \mu B \beta$ \\
-\\
-Die Energie des Systems ist nun gegebn durch:
+Annahmen:
+\begin{itemize}
+ \item periodische Randbedingungen, $S_{N+1} \equiv S_1$
+ \item Translationsinvarianz, $J_{i,i+1} \equiv J$
+\end{itemize}
+Abkuerzung: $K = \beta J$, $h = \mu B \beta$
+\end{slide}
+
+\begin{slide}
+Energie des Systems:
 \[
  E = -J \sum_{i=1}^{N} S_i S_{i+1} - \mu B_0 \sum_{i=1}^{N} S_i
 \]
 \[
  E = -J \sum_{i=1}^{N} S_i S_{i+1} - \mu B_0 \sum_{i=1}^{N} S_i
 \]
-Die Magnetisierung entspricht dem Erwartungswert des magnetischen Moments an einem Gitterplatz:
-\[
- M = <S_1>
-\]
-Es gibt $2^N$ moegliche Spinzustaende. Die Zustandssumme lautet:
+% Magnetisierung:
+\[
+ M = <S_1>
+\]
+Zustandssumme:
 \[
  Z = \sum_{S_1} \sum_{S_2} \ldots \sum_{S_N} e^{\beta ( K \sum_{i=1}^{N} S_i S_{i,i+1} + \frac{1}{2} h \sum_{i=1}^{N} S_i + S_{i,i+1} ) }
 \]
 \[
  Z = \sum_{S_1} \sum_{S_2} \ldots \sum_{S_N} e^{\beta ( K \sum_{i=1}^{N} S_i S_{i,i+1} + \frac{1}{2} h \sum_{i=1}^{N} S_i + S_{i,i+1} ) }
 \]
-Die Zustandssumme wird mit Hilfe der Transfer-Matrix-Methode berechnet: \\
+\end{slide}
+
+\begin{slide}
+Bestimmung der Zustandssumme mit Transfer-Matrix-Methode: \\
 \\
 Finde Matrix $\mathbf{T}$ mit fogenden Eigenschaften:
 \[
 \\
 Finde Matrix $\mathbf{T}$ mit fogenden Eigenschaften:
 \[
@@ -195,15 +254,11 @@ Finde Matrix $\mathbf{T}$ mit fogenden Eigenschaften:
  <S_i|\mathbf{T}|S_{i,i+1}> = e^{ K S_i S_{i,i+1} + \frac{h}{2} ( S_i + S_{i,i+1} )} \\
  \\
  \textrm{also:} \\
  <S_i|\mathbf{T}|S_{i,i+1}> = e^{ K S_i S_{i,i+1} + \frac{h}{2} ( S_i + S_{i,i+1} )} \\
  \\
  \textrm{also:} \\
- <1|\mathbf{T}|1> = e^{K+h} \\
- <-1|\mathbf{T}|-1> = e^{K-h} \\
- <1|\mathbf{T}|-1> = <-1|\mathbf{T}|1> = e^{-K} \\
- \\
- wobei: \\
- \begin{array}{ll}
-  |S_i = +1> = \left( \begin{array}{c} 1 \\ 0 \end{array} \right) \, \textrm{,} &
-  |S_i = -1> = \left( \begin{array}{c} 0 \\ 1 \end{array} \right)
- \end{array}
+ \displaystyle <1|\mathbf{T}|1> = e^{K+h} \\[2mm]
+ \displaystyle <-1|\mathbf{T}|-1> = e^{K-h} \\[2mm]
+ \displaystyle <1|\mathbf{T}|-1> = <-1|\mathbf{T}|1> = e^{-K} \\[2mm]
+ \displaystyle |S_i = +1> = \left( \begin{array}{c} 1 \\ 0 \end{array} \right) \, \textrm{,}
+ \quad |S_i = -1> = \left( \begin{array}{c} 0 \\ 1 \end{array} \right)
 \end{array}
 \]
 Die Matrix muss also wie folgt aussehen:
 \end{array}
 \]
 Die Matrix muss also wie folgt aussehen:
@@ -217,43 +272,58 @@ Die Matrix muss also wie folgt aussehen:
  \right)
  \qquad \textrm{Transfer-Matrix}
 \]
  \right)
  \qquad \textrm{Transfer-Matrix}
 \]
-Damit laesst sich die Zustandssumme neu schreiben:
+\end{slide}
+
+\begin{slide}
+Zustandssumme:
 \[
  \begin{array}{ll}
 \[
  \begin{array}{ll}
- Z & = \sum_{S_1} \sum_{S_2} \ldots \sum_{S_N} <S_1|\mathbf{T}|S_2> <S_2|\mathbf{T}|S_3> \ldots <S_{N-1}|\mathbf{T}|S_N> <S_N|\mathbf{T}|S_1> \\
-   & = \sum_{S_1} <S_1|\mathbf{T}^N|S_1> \\
  & = \textrm{Sp} \, \mathbf{T}^N
+ \displaystyle Z & \displaystyle = \sum_{S_1} \sum_{S_2} \ldots \sum_{S_N} <S_1|\mathbf{T}|S_2> <S_2|\mathbf{T}|S_3> \ldots <S_{N-1}|\mathbf{T}|S_N> <S_N|\mathbf{T}|S_1> \\[2mm]
+ \displaystyle  & \displaystyle = \sum_{S_1} <S_1|\mathbf{T}^N|S_1> \\[2mm]
\displaystyle  & \displaystyle = \textrm{Sp} \, \mathbf{T}^N
  \end{array}
 \]
  \end{array}
 \]
-Wegen der Vollstaendigkeit der Spinzustaende ist $\mathbf{T}$ diagonalisierbar und die Spur Darstellungsunabhaengig. Aus $\textrm{det} ( \mathbf{T} - \lambda \mathbf{1} ) = 0$, erhaelt man folgende Eigenwerte:
+\begin{itemize}
+\item Trick: Vollstaendigkeit der Spinzustaende
+\item $\mathbf{T}$ diagonalisierbar, Spur Darstellungsunabhaengig
+\item Eigenwerte: $\textrm{det} ( \mathbf{T} - \lambda \mathbf{1} ) = 0$
+\end{itemize}
 \[
  \lambda_{\pm} = e^{K} ( \cosh h \pm \sqrt{\sinh^2 h + e^{-4K}} )
 \]
 \[
  \lambda_{\pm} = e^{K} ( \cosh h \pm \sqrt{\sinh^2 h + e^{-4K}} )
 \]
-Daraus folgt:
 \[
  \textrm{Sp} \, \mathbf{T}^N = \lambda_+^N + \lambda_-^N = Z
 \]
 \[
  \textrm{Sp} \, \mathbf{T}^N = \lambda_+^N + \lambda_-^N = Z
 \]
+\end{slide}
+
+\begin{slide}
 Fuer den Fall $B_0 = 0$ gilt:
 \[
  \begin{array}{l}
 Fuer den Fall $B_0 = 0$ gilt:
 \[
  \begin{array}{l}
-  \lambda_{\pm} = e^K \pm e^{-K} \\
-  Z = 2^N \cosh^N (K) + 2^N \sinh^N (K) = 2^N \cosh^N (K) (1 + \tanh^N (K)) \stackrel{N >> 1}{\longrightarrow} 2^N \cosh^N (K) \\
-  F = -k_B T \, \textrm{ln} \, Z \stackrel{N >> 1}{\longrightarrow} -N k_B T \, \textrm{ln} \, (2 \cosh (\beta J))
+  \displaystyle \lambda_{\pm} = e^K \pm e^{-K} \\[2mm]
+  \displaystyle Z = 2^N \cosh^N (K) + 2^N \sinh^N (K) = 2^N \cosh^N (K) (1 + \tanh^N (K)) \stackrel{N >> 1}{\longrightarrow} 2^N \cosh^N (K) \\[2mm]
+  \displaystyle F = -k_B T \, \textrm{ln} \, Z \stackrel{N >> 1}{\longrightarrow} -N k_B T \, \textrm{ln} \, (2 \cosh (\beta J))
  \end{array}
 \]
  \end{array}
 \]
-Dabei wurde verwendet, dass $\lambda_+^N$ im thermodynamischen Limes viel groesser ist als $\lambda_-^N$. \\
-Fuer die Magnetisierung mit Magnetfeld gilt:
+weil $\lambda_+^N$ viel groesser als $\lambda_-^N$. (thermodynamischer Limes)\\
+Magnetisierung:
 \[
  \begin{array}{ll}
 \[
  \begin{array}{ll}
-  M & = \frac{1}{Z} \sum_{\{S\}} (\sum_{i} \mu S_i) e^{-\beta H} \\
-    & = \frac{1}{\beta} (\frac{\partial}{\partial{B_0}} \, \textrm{ln} \, Z) \\
-    & \stackrel{N >> 1}{\longrightarrow} \frac{N}{\beta \lambda_+} \frac{\partial{\lambda_+}}{\partial{B_0}} \\
-    & = N \mu \frac{\sinh (\beta \mu B_0)}{\sqrt{\cosh^2 (\beta \mu B_0) - 2e^{-2 \beta J} \sinh (2 \beta J)}}
-  
+  \displaystyle M & = \frac{1}{Z} \sum_{\{S\}} (\sum_{i} \mu S_i) e^{-\beta H} \\[2mm]
+  \displaystyle & = \frac{1}{\beta} (\frac{\partial}{\partial{B_0}} \, \textrm{ln} \, Z) \\[2mm]
+  \displaystyle & \stackrel{N >> 1}{\longrightarrow} \frac{N}{\beta \lambda_+} \frac{\partial{\lambda_+}}{\partial{B_0}} \\[2mm]
+  \displaystyle & \displaystyle = N \mu \frac{\sinh (\beta \mu B_0)}{\sqrt{\cosh^2 (\beta \mu B_0) - 2e^{-2 \beta J} \sinh (2 \beta J)}}
  \end{array}
 \]
  \end{array}
 \]
-Folgende Abbidlung zeigt die Magnetisierung in Abhaengigkeit vom Magnetfeld. Die Magnetisierung verschwindet fuer alle Temperaturen wenn kein Magnetfeld vorhanden ist. Fuer sehr grosse Magnetfelder saettigt sie.
-\\
+\end{slide}
+
+\begin{slide}
+Abbidlung: 
+\begin{itemize}
+\item Magnetisierung in Abhaengigkeit vom Magnetfeld
+\item Magnetisierung verschwindet fuer alle Temperaturen wenn kein Magnetfeld vorhanden ist
+\item S"attigung f"ur gro"se Magnetfelder
+\end{itemize}
 \setlength{\unitlength}{2cm}
 \begin{picture}(6,4)(-3,-2)
  \put(-2.5,0){\vector(1,0){5}}
 \setlength{\unitlength}{2cm}
 \begin{picture}(6,4)(-3,-2)
  \put(-2.5,0){\vector(1,0){5}}
@@ -265,55 +335,68 @@ Folgende Abbidlung zeigt die Magnetisierung in Abhaengigkeit vom Magnetfeld. Die
  \qbezier(0,0)(0.8853,0.8853)(2,0.9640)
  \qbezier(0,0)(-0.8853,-0.8853)(-2,-0.9640)
 \end{picture}
  \qbezier(0,0)(0.8853,0.8853)(2,0.9640)
  \qbezier(0,0)(-0.8853,-0.8853)(-2,-0.9640)
 \end{picture}
-\\
+\end{slide}
+
+\begin{slide}
 Erkenntnis:\\
 \begin{itemize}
 \item magnetisches Moment verschwindet fuer alle endlichen Temperaturen wenn $B_0 = 0$
 \item es gibt keinen Phasenuebergang fuer das eindimensionale Isingmodell fur $T>0$
 \end{itemize}
 Erkenntnis:\\
 \begin{itemize}
 \item magnetisches Moment verschwindet fuer alle endlichen Temperaturen wenn $B_0 = 0$
 \item es gibt keinen Phasenuebergang fuer das eindimensionale Isingmodell fur $T>0$
 \end{itemize}
-F"ur $T=0$ kann obere Approximation nichtmehr verwendet werden, da gilt:
+F"ur $T=0$:
 \[ 
 \[ 
- \lim_{T \rightarrow 0} \frac{\lambda_+}{\lambda_-}=1
+ \lim_{T \rightarrow 0} \frac{\lambda_+}{\lambda_-}=1 \, \textrm{obere Approximation nichtmehr g"ultig)}
 \]
 \]
-Mann kann zeigen, da"s bei $B_0=T=0$ ein Phasen"ubergagng liegt (Korrelationsl"ange geht gegen unendlich), und eine spontane Magnetisierung existiert. Kritische Exponenten:
+Phasen"ubergang bei $B_0=T=0$ (Korrelationsl"ange geht gegen unendlich)\\
+Kritische Exponenten:
 \[
  \alpha = 1 \qquad \beta = 0 \qquad \gamma = 1
 \]
 \[
  \alpha = 1 \qquad \beta = 0 \qquad \gamma = 1
 \]
+\end{slide}
 
 
-\subsection{2-dimensionale L"osung}
-W"ahrend das eindimensionale Modell noch relativ leicht zu l"osen war, und deshalb hier detailliert beschrieben wurde, ist das zweidimensionale h"ochst nichttrivial. Es wird auf eine genau L"osung verzichtet. Tats"achlich ist das einzige Problem die Diagonalisierung einer $2^N \times 2^N$ - Matrix (wieder Transfer-Matrix-Methode). Eine L"osung wird nur ohne vorhandenes Magnetfeld gefunden.\\
-\\
+\begin{slide}
+\slideheading{L"osung f"ur $d=2$}
+\begin{itemize}
+\item TFM analog $d=1$ L"osung
+\item $B=0$ \, \textrm{L"osung nur ohne vorhandenes Magnetfeld}
+\end{itemize}
 Hamiltonian:
 \[
  H = -J \sum_{(i,j)} (S_{i,j} S_{i+1,j} + S_{i,j} S_{i,j+1}) - \mu B_0 \sum_{i,j} S_{i,j}\\
 \]
 Hamiltonian:
 \[
  H = -J \sum_{(i,j)} (S_{i,j} S_{i+1,j} + S_{i,j} S_{i,j+1}) - \mu B_0 \sum_{i,j} S_{i,j}\\
 \]
-Dabei geben die Indizes der Spins deren Punkt im Gitter an. Dies schreiben wir k"urzer
+Indizes $\equiv$ Gitterpunkte der Spins. Abk"urzung:
 \[
  H = \sum_{j=1}^{N} \Big( E(\mu_j,\mu_{j+1}) + E(\mu_j) \Big)
 \]
 wobei
 \[
 \begin{array}{ll}
 \[
  H = \sum_{j=1}^{N} \Big( E(\mu_j,\mu_{j+1}) + E(\mu_j) \Big)
 \]
 wobei
 \[
 \begin{array}{ll}
- E(\mu_j,\mu_k) & \equiv - \sum_{i=1}^N S_{i,j} S_{i,k} \\
- E(\mu_j)       & \equiv - J \sum_{i=1}^N S_{i,j} S_{i+1,j} - \mu B_0 \sum_{i,j} S_j \\
- \mu_j          & \equiv \{S_{1,j},\ldots,S_{N,j}\}
+ \displaystyle E(\mu_j,\mu_k) & \displaystyle \equiv - \sum_{i=1}^N S_{i,j} S_{i,k} \\[2mm]
+ \displaystyle E(\mu_j)       & \displaystyle \equiv - J \sum_{i=1}^N S_{i,j} S_{i+1,j} - \mu B_0 \sum_{i,j} S_j \\[2mm]
+ \displaystyle \mu_j          & \displaystyle \equiv \{S_{1,j},\ldots,S_{N,j}\}
 \end{array}
 \]
 \end{array}
 \]
-Damit bestimmen wir analog zum eindimensionalen Fall eine Transfer-Matrix $\mathbf{T}$, mit Matrixelementen:
+\end{slide}
+
+\begin{slide}
+Transfer-Matrix $\mathbf{T}$, mit Matrixelementen:
 \[
 \[
- <\mu_j|\mathbf{T}|\mu_k> = e^{- \beta \Big( E(mu_j,\mu_k) + E(\mu_j) \Big)}
+ <\mu_j|\mathbf{T}|\mu_k> = e^{- \beta \Big( E(\mu_j,\mu_k) + E(\mu_j) \Big)}
 \]
 \]
-Dies ist eine $2^N \times 2^N$ - Matrix, die es wie erw"ahnt zu diagonalisieren gilt. Analog zum $d=1$ Fall gilt:
+$2^N \times 2^N$ - Matrix, Diagonalisierung. Analog zum $d=1$ Fall gilt:
 \[
 \[
- Z = \textrm{Sp} \, mathbf{T}^N
+ Z = \textrm{Sp} \, \mathbf{T}^N
 \]
 \]
-Diesen Schritt kann man sich zum Beispiel in [\ref{lit7}] genauer anschauen. Im Folgenden Werden nur die Endresultate betrachtet.\\
-\\
-F"ur die freie Energie pro Spin $f = \lim_{N \to \infty} \frac{1}{N} (-k_B T \, \textrm{ln} \, Z)$ erh"alt man
+[\ref{lit7}] Kerson Huang, Statistical mechanics
+\end{slide}
+
+\begin{slide}
+freie Energie pro Spin $f = \lim_{N \to \infty} \frac{1}{N} (-k_B T \, \textrm{ln} \, Z)$: 
 \[
  f = -k_B T \textrm{ln} \, \Big( 2 \cosh(2 \beta J) \Big) - \frac{k_B T}{2 \pi} \int_{0}^{\pi} d\phi \, \textrm{ln} \, \frac{1}{2} \Bigg( 1 + \sqrt{1 - K^2 \sin^2 \phi} \Bigg)
 \]
 \[
  f = -k_B T \textrm{ln} \, \Big( 2 \cosh(2 \beta J) \Big) - \frac{k_B T}{2 \pi} \int_{0}^{\pi} d\phi \, \textrm{ln} \, \frac{1}{2} \Bigg( 1 + \sqrt{1 - K^2 \sin^2 \phi} \Bigg)
 \]
-mit $K = \frac{2}{\cosh (2 \beta J) \coth (2 \beta J)}$, und demnach f"ur die Magnetisierung:
+mit $\displaystyle K = \frac{2}{\cosh (2 \beta J) \coth (2 \beta J)}$\\
+Magnetisierung:
 \[
  m = \left\{
  \begin{array}{ll}
 \[
  m = \left\{
  \begin{array}{ll}
@@ -321,47 +404,65 @@ mit $K = \frac{2}{\cosh (2 \beta J) \coth (2 \beta J)}$, und demnach f"ur die Ma
   0 & : T > T_C
  \end{array} \right.
 \]
   0 & : T > T_C
  \end{array} \right.
 \]
-F"ur den kritischen Exponenten $\beta$ gilt also $\beta = \frac{1}{8}$. Als Bedingung f"ur die kritische Temperatur erh"alt man:
-\[
+kritische Temperatur:
+\[ 
  2 \tanh^2 (2 \beta J) = 1 \, \textrm{, also} \, k_B T_C \approx 2.269185 \cdot J
 \]
  2 \tanh^2 (2 \beta J) = 1 \, \textrm{, also} \, k_B T_C \approx 2.269185 \cdot J
 \]
-In der N"ahe von $T=T_C$ erkennt man eine logarithmische Divergenz der spezifischen W"arme.
+spezifische W"arme: (logarithmische Divergenz)
 \[
 \[
-C = k_B \frac{2}{\pi} \Big( \frac{2 J}{k_B T_C} \Big) \Bigg( - \textrm{ln} \, \Big( 1 - \frac{T}{T_C} \Big) + \textrm{ln} \, \Big( \frac{k_B T_C}{2 J} \Big) - \Big( 1 + \frac{\pi}{4} \Big) \Bigg)
+C = k_B \frac{2}{\pi} \Big( \frac{2 J}{k_B T_C} \Big) \Bigg( - \textrm{ln} \, \Big( 1 - \frac{T}{T_C} \Big)
++ \textrm{ln} \, \Big( \frac{k_B T_C}{2 J} \Big) - \Big( 1 + \frac{\pi}{4} \Big) \Bigg)
 \]
 \]
-Damit ist der kritische Exponent $\alpha = 0$.\\
-\\
+kritische Exponenten:\\
+$\beta = \frac{1}{8}$ \\
+$\alpha = 0$
+\end{slide}
+
+\begin{slide}
 Fazit:
 \begin{itemize}
 Fazit:
 \begin{itemize}
-\item es existiert ein Phasenuebergang zweiter Ordnung
-\item auch ohne vorhandenes Magnetfeld hat der Ising Ferromagnet eine spontane Magnetisierung wenn $T < T_C$
+\item Phasenuebergang zweiter Ordnung
+\item spontane Magnetisierung wenn $T < T_C$ ohne vorhandenes Magnetfeld
 \end{itemize}
 \end{itemize}
+\end{slide}
 
 
-\subsection{3-dimensionale L"osung}
+\begin{slide}
+\slideheading{L"osung f"ur $d=3$}
 Das dreidimensionale Modell kann bis heute nicht exakt analytisch gel"ost werden. Approximationen und Monte Carlo Simulationen liefern jedoch ueberzeugende Resultate. Man erwartet von der analytisch exakten L"osung keine weiteren Informationen mehr.\\
 \\
 Das dreidimensionale Ising Modell zeigt Phasenueberg"ange.
 Das dreidimensionale Modell kann bis heute nicht exakt analytisch gel"ost werden. Approximationen und Monte Carlo Simulationen liefern jedoch ueberzeugende Resultate. Man erwartet von der analytisch exakten L"osung keine weiteren Informationen mehr.\\
 \\
 Das dreidimensionale Ising Modell zeigt Phasenueberg"ange.
+\end{slide}
 
 
+\begin{slide}
 \section{Monte Carlo Simulation}
 \section{Monte Carlo Simulation}
-Im Folgenden soll gezeigt werden wie man durch sogenannte Monte Carlo Simulationen das Ising Modell simuliert.\\
+\end{slide}
+
+\begin{slide}
+Simulationen das Ising Modell durch Monte Carlo Simulation\\
 \\
 Gesucht sei der Erwartungswert $<A>$.
 \[
 \begin{array}{l}
 \\
 Gesucht sei der Erwartungswert $<A>$.
 \[
 \begin{array}{l}
- <A> = \sum_i p_i A_i \, \textrm{, wobei} \\
- p_i = \frac{e^{- \beta E_i}}{\sum_j e^{\beta E_j}} \, \textrm{Boltzmann Wahrscheinlichkeitsverteilung} \\
- E_i \, \textrm{Energie im Zustand i}
+ \displaystyle <A> = \sum_i p_i A_i \, \textrm{, wobei} \\[2mm]
+ \displaystyle p_i = \frac{e^{- \beta E_i}}{\sum_j e^{\beta E_j}} \, \textrm{Boltzmann Wahrscheinlichkeitsverteilung} \\[2mm]
\displaystyle E_i \, \textrm{Energie im Zustand i}
 \end{array}
 \]
 \end{array}
 \]
-Anstatt ueber alle Zust"ande zu summieren, greift man nur einige zuf"allige Zust"ande auf, deren Wahrscheinlichkeit idealerweise nat"urlich der Boltzmannverteilung entspricht.
+\end{slide}
+
+\begin{slide}
+Anstatt ueber alle Zust"ande zu summieren, greift man nur einige zuf"allige Zust"ande auf, deren Wahrscheinlichkeit idealerweise nat"urlich der Boltzmannverteilung entspricht (importance sampling).
 \[
  <A>_{est} = \frac{1}{N} \sum_{i=1}^{N} A(i)
 \]
 \[
  <A>_{est} = \frac{1}{N} \sum_{i=1}^{N} A(i)
 \]
-$N$ entspricht hierbei der Anzahl der Itterationen in der Computersimulation.
+$N$ entspricht hierbei der Anzahl der Iterationen in der Computersimulation.
 \begin{itemize}
  \item $P(A,t)$ sei die Wahrscheinlichkeit der Konfiguration $A$ zur Zeit $t$
 \begin{itemize}
  \item $P(A,t)$ sei die Wahrscheinlichkeit der Konfiguration $A$ zur Zeit $t$
- \item $W(A \rightarrow B)$ sei Wahrscheinlichkeit pro Zeiteinheit, da"s dir Konfiguration von $A$ nach $B$ wechselt
+ \item $W(A \rightarrow B)$ sei Wahrscheinlichkeit pro Zeiteinheit, da"s die Konfiguration von $A$ nach $B$ wechselt
 \end{itemize}
 \end{itemize}
+\end{slide}
+
+\begin{slide}
 Damit gilt:
 \[
  P(A,t+1) = P(A,t) + \sum_B \Big( W(B \rightarrow A) P(B,t) - W(A \rightarrow B) P(A,t) \Big)
 Damit gilt:
 \[
  P(A,t+1) = P(A,t) + \sum_B \Big( W(B \rightarrow A) P(B,t) - W(A \rightarrow B) P(A,t) \Big)
@@ -369,13 +470,17 @@ Damit gilt:
 und f"ur gro"se $t$ ist dir willk"urliche Anfangskonfiguration vergessen, $P(A,t) \rightarrow p(A)$.\\
 Eine Bedingung f"ur eine zeitunabh"angige Wahrscheinlichkeitsverteilung ist:
 \[
 und f"ur gro"se $t$ ist dir willk"urliche Anfangskonfiguration vergessen, $P(A,t) \rightarrow p(A)$.\\
 Eine Bedingung f"ur eine zeitunabh"angige Wahrscheinlichkeitsverteilung ist:
 \[
- W(A \rightarrow B) P(A,t) = W(B \rightarrow A) P(B,t)
+ W(A \rightarrow B) P(A,t) = W(B \rightarrow A) P(B,t) \qquad \textrm{(detailed balance)}
 \]
 und somit gilt:
 \[
  \frac{W(A \rightarrow B)}{W(B \rightarrow A)} = \frac{p(B)}{p(A)} = \frac{e^{- \beta E(B)}}{e^{- \beta E(A)}} = e^{- \beta \delta E}
 \]
 \]
 und somit gilt:
 \[
  \frac{W(A \rightarrow B)}{W(B \rightarrow A)} = \frac{p(B)}{p(A)} = \frac{e^{- \beta E(B)}}{e^{- \beta E(A)}} = e^{- \beta \delta E}
 \]
-Eine Realisierung einer solchen Boltzmannverteilung bietet der Metropolis Algorithmus [\ref{lit4}].\\
+\end{slide}
+
+\begin{slide}
+Eine Realisierung einer solchen Boltzmannverteilung bietet der Metropolis Algorithmus\\
+[\ref{lit4}] http://www.npac.syr.edu/users/gcf/slitex/CPS713MonteCarlo96/index.html
 \[
  W(A \rightarrow B) = \left\{
  \begin{array}{ll}
 \[
  W(A \rightarrow B) = \left\{
  \begin{array}{ll}
@@ -388,12 +493,16 @@ Der Pseudocode eines Programms k"onnte nun wie folgt aussehen:
 \item Gehe alle Gitterplaetze durch
 \item Berechne $\delta E$ fuer Spinflip (Naechste Nachbarn anschauen)
 \item Wenn $\delta E < 0$ flip, ansonsten nur wenn Zufallszahl kleiner $e^{\frac{-\delta E}{k_B T}}$
 \item Gehe alle Gitterplaetze durch
 \item Berechne $\delta E$ fuer Spinflip (Naechste Nachbarn anschauen)
 \item Wenn $\delta E < 0$ flip, ansonsten nur wenn Zufallszahl kleiner $e^{\frac{-\delta E}{k_B T}}$
-\item Spins aufsummieren, dies entspricht der Magnetisierung (nach genuegend vielen Itterationen ($N^3$))
+\item Spins aufsummieren, dies entspricht der Magnetisierung (nach genuegend vielen Iterationen ($N^3$))
 \end{itemize}
 \end{itemize}
+\end{slide}
 
 
+\begin{slide}
 \section{Anwendungen}
 \section{Anwendungen}
-\begin{itemize}
-\item Spingl"aser [\ref{lit8}]
+\end{slide}
+
+\begin{slide}
+Spingl"aser ([\ref{lit8}] W. Kinzel, Spingl"aser, Optimierung und Ged"achtnis)
  \begin{itemize}
  \item Betrifft: magnetische Legierungen (Bsp.: $Au_{1-x}Fe_x$)
  \item Beobachtungen:
  \begin{itemize}
  \item Betrifft: magnetische Legierungen (Bsp.: $Au_{1-x}Fe_x$)
  \item Beobachtungen:
@@ -408,12 +517,15 @@ Der Pseudocode eines Programms k"onnte nun wie folgt aussehen:
   \item Hamilton: $H = - \sum J_{ij} S_i S_j - \mu B_0 \sum S_i$, wobei die $J_{ij}$ zufaellige, symmetrisch um $0$ verteilte Kopplung darstellt
   \end{itemize}
  \end{itemize}
   \item Hamilton: $H = - \sum J_{ij} S_i S_j - \mu B_0 \sum S_i$, wobei die $J_{ij}$ zufaellige, symmetrisch um $0$ verteilte Kopplung darstellt
   \end{itemize}
  \end{itemize}
-\item Spingl"aser: Optimierung und Ged"achtnis [\ref{lit8}]
+\end{slide}
+
+\begin{slide}
+Spingl"aser: Optimierung und Ged"achtnis [\ref{lit8}]
  \begin{itemize}
   \item Traveling Salesman Problem:
    \begin{itemize}
  \begin{itemize}
   \item Traveling Salesman Problem:
    \begin{itemize}
-   \item "Aufheizen" des Systems, Wegstrecken bekommen gleiche Gewichtung
-   \item "Abk"uhlen des Systems, Zustand niedrigster Energie stellt sich ein, der ideale Weg?
+   \item \dq Aufheizen \dq des Systems, Wegstrecken bekommen gleiche Gewichtung
+   \item \dq Abk"uhlen \dq des Systems, Zustand niedrigster Energie stellt sich ein, der ideale Weg?
    \end{itemize}
   \item Ged"achtnis:
   \begin{itemize}
    \end{itemize}
   \item Ged"achtnis:
   \begin{itemize}
@@ -436,6 +548,10 @@ Der Pseudocode eines Programms k"onnte nun wie folgt aussehen:
    \end{itemize}
   \end{itemize}
  \end{itemize}
    \end{itemize}
   \end{itemize}
  \end{itemize}
+\end{slide}
+
+\begin{slide}
+\begin{itemize}
 \item Ghetto Formationen [\ref{lit5}]\\
  \[
   \begin{array}{ll}
 \item Ghetto Formationen [\ref{lit5}]\\
  \[
   \begin{array}{ll}
@@ -459,9 +575,13 @@ Der Pseudocode eines Programms k"onnte nun wie folgt aussehen:
  \item duopoly markets
  \end{itemize}
 \end{itemize}
  \item duopoly markets
  \end{itemize}
 \end{itemize}
+\end{slide}
 
 
-\appendix
+\begin{slide}
 \section{Quellen}
 \section{Quellen}
+\end{slide}
+
+\begin{slide}
 \begin{enumerate}
 \item \label{lit1} W. Nolting, Grundkurs: Statistische Physik, Band 6
 \item \label{lit2} Rodney J. Baxter, Exactly Solved Models in Statistical Mechanics
 \begin{enumerate}
 \item \label{lit1} W. Nolting, Grundkurs: Statistische Physik, Band 6
 \item \label{lit2} Rodney J. Baxter, Exactly Solved Models in Statistical Mechanics
@@ -472,5 +592,6 @@ Der Pseudocode eines Programms k"onnte nun wie folgt aussehen:
 \item \label{lit7} Kerson Huang, Statistical mechanics
 \item \label{lit8} W. Kinzel, Spingl"aser, Optimierung und Ged"achtnis
 \end{enumerate}
 \item \label{lit7} Kerson Huang, Statistical mechanics
 \item \label{lit8} W. Kinzel, Spingl"aser, Optimierung und Ged"achtnis
 \end{enumerate}
+\end{slide}
 
 \end{document}
 
 \end{document}